Fórum de Matemática
DÚVIDAS? Nós respondemos!

Um Fórum em Português dedicado à Matemática
Data/Hora: 28 mar 2024, 10:00

Os Horários são TMG [ DST ]




Fazer Nova Pergunta Responder a este Tópico  [ 3 mensagens ] 
Autor Mensagem
MensagemEnviado: 26 jun 2016, 15:23 
Offline

Registado: 17 mai 2016, 04:19
Mensagens: 13
Localização: Rio de Janeiro
Agradeceu: 0 vez(es)
Foi agradecido: 0 vez(es)
Bom dia amiguinhos

Não estou conseguindo solucionar essa questão de indução:

Anexo:
QuestaoInduçao.png
QuestaoInduçao.png [ 22.08 KiB | Visualizado 1020 vezes ]


Poderiam me ajudar por gentileza?

Grata desde já

;)


Topo
 Perfil  
 
MensagemEnviado: 27 jun 2016, 13:03 
Offline

Registado: 17 jan 2013, 13:36
Mensagens: 2487
Localização: Lisboa
Agradeceu: 31 vezes
Foi agradecido: 1049 vezes
Note que:
\(S_n=a_1 + qa_1 + q^2 a_1 + \cdots q^{n-1} a_1\)
\(q S_n = q a_1 + q^2 a_2 + \cdots q^n a_1\)

então

\(q S_n - S_n =a_1 q^n - a_1 \Leftrightarrow S_n= \frac{a_1 q^n -a_1}{q-1}\)


Topo
 Perfil  
 
MensagemEnviado: 27 jun 2016, 16:53 
Offline

Registado: 19 Oct 2015, 13:34
Mensagens: 929
Localização: Rio de Janeiro
Agradeceu: 9 vezes
Foi agradecido: 274 vezes
base:
\(\left \{ 2, 4, 8 \right \}\)
\(S_n=\frac{a_1q^n-a1}{q-1}\)
\(S_3=\frac{2.2^3-2}{2-1}\)
\(S_3=14\)

hipótese:
funciona para:
\(\forall n\in \mathbb{N}, n\geq 1
e,
q \in \mathbb{Q}, {0}\neq q\neq 1\)
e,
\(q=\frac{a_n}{a_{n-1}}\)

tese:
\(S_n=\frac{a_1q^n-a1}{q-1}\)

demonstração:
feita pelo Sobolev

_________________
Vivemos em um mundo onde toda informação é falsa até que se prove o contrário.
A Verdade está a caminho.


Topo
 Perfil  
 
Mostrar mensagens anteriores:  Ordenar por  
Fazer Nova Pergunta Responder a este Tópico  [ 3 mensagens ] 

Os Horários são TMG [ DST ]


Quem está ligado:

Utilizadores a ver este Fórum: Nenhum utilizador registado e 34 visitantes


Criar perguntas: Proibído
Responder a perguntas: Proibído
Editar Mensagens: Proibído
Apagar Mensagens: Proibído
Enviar anexos: Proibído

Pesquisar por:
Ir para:  
cron